Đến nội dung

dogsteven nội dung

Có 1000 mục bởi dogsteven (Tìm giới hạn từ 18-05-2020)



Sắp theo                Sắp xếp  

#567445 Topic tổng hợp một số bất đẳng thức trong kì thi MO các nước

Đã gửi bởi dogsteven on 22-06-2015 - 16:05 trong Bất đẳng thức - Cực trị

Bài 138(tự sáng tác)(1 bài tập chặt hơn bài 136):  Cho a,b,c >0:ab+ca+bc=1. CMR:

$\frac{1}{a^2+b^2+bc}+\frac{1}{b^2+c^2+ca}+\frac{1}{c^2+a^2+ab}\leq 3(a^2+b^2+c^2)$

$VT=\sum \dfrac{c^2+a^2+bc}{(a^2+b^2+bc)(c^2+a^2+bc)}\leqslant \sum \dfrac{c^2+a^2+bc}{(ab+bc+ca)^2}=2\sum a^2+\sum ab\leqslant 3\sum a^2$




#567450 Topic tổng hợp một số bất đẳng thức trong kì thi MO các nước

Đã gửi bởi dogsteven on 22-06-2015 - 16:12 trong Bất đẳng thức - Cực trị

Dám hỏi dấu bằng xảy ra khi nào???

$a=b=1, c=0$ và các hoán vị.




#567215 Topic tổng hợp một số bất đẳng thức trong kì thi MO các nước

Đã gửi bởi dogsteven on 21-06-2015 - 07:03 trong Bất đẳng thức - Cực trị

Bài này thực sự là đặc chưng cho phương pháp tiếp tuyến, ta giải như sau:

1) Áp dụng bất đẳng thức AM-GM và giả thuyết thì chúng ta sẽ có BĐT đã cho tương đương BĐT sau:

$\sum \frac{1}{a^2-6a+13}\leq \frac{3}{8}$

Thật vậy lưu ý bất đẳng thức phụ sau:

$\frac{1}{a^2-6a+13}\leq \frac{a+1}{16}$

tương tự như thế suy ra$\sum \frac{1}{a^2-6a+13}\leq \frac{a+b+c+3}{16}=\frac{3}{8}$

từ đó ta có đpcm

Giả sử $a\geqslant b\geqslant c$. Bất đẳng thức trên tương đương với: $\sum \dfrac{(a+b)^2}{(a+b)^2+\dfrac{2(a+b)^2}{a^2+b^2}}\geqslant \dfrac{3}{2}$

Áp dụng bất đẳng thức Cauchy-Schwarz: $VT\geqslant \sum \dfrac{4(a+b+c)^2}{\sum (a+b)^2+\sum \dfrac{2(a+b)^2}{a^2+b^2}}$

Do đó ta cần chứng minh: $\sum (a+b)^2+\sum \dfrac{2(a+b)^2}{a^2+b^2}\leqslant 24$

$\Leftrightarrow \dfrac{4}{3} (a+b+c)^2-\sum (a+b)^2+12-\sum \dfrac{2(a+b)^2}{a^2+b^2}\geqslant 0\Leftrightarrow \sum (a-b)^2\left(\dfrac{6}{a^2+b^2}-1\right)\geqslant 0$

Nếu $a^2+b^2\leqslant 6$ thì bất đẳng thức đúng. Nếu $a^2+b^2\geqslant  6$ thì:

$\sum \dfrac{1}{a^2+b^2+2}\leqslant \dfrac{1}{8}+\dfrac{1}{a^2+2}+\dfrac{1}{b^2+6} \leqslant \dfrac{1}{8}+\dfrac{1}{8-b^2}+\dfrac{1}{b^2+2}\leqslant \dfrac{1}{8}+\dfrac{5}{8}=\dfrac{3}{4}$




#567137 Topic tổng hợp một số bất đẳng thức trong kì thi MO các nước

Đã gửi bởi dogsteven on 20-06-2015 - 17:00 trong Bất đẳng thức - Cực trị

Bài 130(IMO 2001): Cho a,b,c>0. CMR:

$\sum \frac{a}{\sqrt{a^2+8bc}}\geq 1$

Có thể dùng Holder nhưng không thích, ta dùng dao mổ trâu :v

$x=\sqrt{\dfrac{a^2}{a^2+8bc}}, y=..., z=...$, ta cần chứng minh $x+y+z\geqslant 1$

Chú ý rằng $(1-x^2)(1-y^2)(1-z^2)=8^3x^2y^2z^2$, nếu $x+y+z<1$ thì:

$8^3x^2y^2z^2>(y+z)(z+x)(x+y)(y+z+2x)(z+x+2y)(x+y+2z)$

Bất đẳng thức hiển nhiên sai theo AM-GM




#567136 Topic tổng hợp một số bất đẳng thức trong kì thi MO các nước

Đã gửi bởi dogsteven on 20-06-2015 - 16:56 trong Bất đẳng thức - Cực trị

Bài 131(Poland MO): Cho a,b,c>0. CMR

$\sqrt{\frac{a^5-a^2+3}{4b+5c}}+\sqrt{\frac{b^5-b^2+3}{4c+5a}}+\sqrt{\frac{c^5-c^2+3}{4a+5b}}\geq 1$

$\prod (a^5-a^3+3)\geqslant \prod (a^2+2)\geqslant (a+b+c)^3$ còn $\prod (4b+5c)\leqslant 3^3(a+b+c)^3$

Do đó $VT\geqslant \sqrt{3}>1$




#567627 Topic tổng hợp một số bất đẳng thức trong kì thi MO các nước

Đã gửi bởi dogsteven on 23-06-2015 - 14:17 trong Bất đẳng thức - Cực trị

Sau đây là lời giải cho bài toán trên:

Không mất tính tổng quát ta giả sử a$\geq$b$\geq$c

Khi đó dễ thấy rằng:$a\geq b\geq c$ $\frac{1}{b^2+8bc+3c^2}\geq \frac{1}{c^2+8ca+3a^2}\geq \frac{1}{a^2+8ab+3b^2}$

Áp dụng bất đẳng thức Chebysev và bất đẳng thức AM-GM, ta có:

$\sum \frac{a}{b^2+4bc+3c^2}\geq \frac{1}{3}(a+b+c)(\sum \frac{1}{b^2+4bc+3c^2})\geq 3(a+b+c)(\frac{1}{4(a+b+c)^2})\geq \frac{1}{4}$ (đpcm)

Nên xem lại.




#567628 Topic tổng hợp một số bất đẳng thức trong kì thi MO các nước

Đã gửi bởi dogsteven on 23-06-2015 - 14:20 trong Bất đẳng thức - Cực trị

Không biết bài này có chưa, nếu có rồi thì anh Đức xóa dùm em

Bài 142(APMO 1996) Cho a,b,c là độ dài 3 cạnh tam giác. Chứng minh rằng:

$\sqrt{a+b-c}+\sqrt{b+c-}+\sqrt{c+a-b}\geq \sqrt{a}+\sqrt{b}+\sqrt{c}$

Đẳng thức xảy ra khi nào

Bị ngược dấu rồi em.

Không mất tính tổng quát, giả sử $a\geslant b\geqslant c$

Dễ thấy $a+b-c\geqslant c+a-b\geqslant b+c-a$

Mà $a+b-c\geqslant a$ và $a+b-c+c+a-b=2a\geqslant a+b$ và $a+b-c+c+a-b+b+c-a=a+b+c$ nên $(a+b-c, c+a-b, b+c-a)\succ (a,b,c)$

Áp dụng bất đẳng thức Karamata cho hàm lõm $f(x)=\sqrt{x}$ cho ta điều phải chứng minh.




#569456 Topic tổng hợp một số bất đẳng thức trong kì thi MO các nước

Đã gửi bởi dogsteven on 02-07-2015 - 15:23 trong Bất đẳng thức - Cực trị

Bài 167(China TST): Cho a,b,c thuộc [0;1]. Tìm GTNN của biểu thức sau:

                              P=$\frac{1}{(b-1)^2+a^2}+\frac{1}{(b-1)^2+c^2}$

Đề có thể sai chăng? Hiển nhiên $P\geqslant 1$, đẳng thức khi $a-1=b=c-1=0$

TST không thể dễ đến vậy




#569453 Topic tổng hợp một số bất đẳng thức trong kì thi MO các nước

Đã gửi bởi dogsteven on 02-07-2015 - 15:16 trong Bất đẳng thức - Cực trị

Bài 165 (Mediterranean Mathematical Competition 2009).Chứng minh rằng với mọi  $a,b,c $ dương ta luôn có:

$$\sum \frac{ab}{a^2+ab+b^2}\leq \frac{a}{2a+b}+\frac{b}{2b+c}+\frac{c}{2c+a}$$

Áp dụng bất đẳng thức Cauchy-Schwarz:

$3-\sum \dfrac{2b}{2b+a}=\sum \dfrac{a}{a+2b}\geqslant \dfrac{(a+b+c)^2}{a^2+b^2+c^2+2ab+2bc+2ca}=1\Rightarrow \sum \dfrac{b}{a+2b}\leqslant 1$

Do đó $\sum \dfrac{a}{2a+b}=\sum \dfrac{a}{2a+b}+\sum \dfrac{b}{2b+a}-1$

Ta sẽ chứng minh: $\dfrac{a}{2a+b}+\dfrac{b}{2b+a}\geqslant \dfrac{ab}{a^2+b^2+ab}+\dfrac{1}{3}$ hay $\dfrac{a^2+b^2+4ab}{(a+2b)(b+2a)}\geqslant \dfrac{a^2+b^2+4ab}{3(a^2+b^2+ab)}\Leftrightarrow (a-b)^2\geqslant 0$




#569440 Topic tổng hợp một số bất đẳng thức trong kì thi MO các nước

Đã gửi bởi dogsteven on 02-07-2015 - 14:18 trong Bất đẳng thức - Cực trị

$P=\left ( \frac{a}{a+b} \right )^3+\left ( \frac{b}{b+c} \right )^3+\left ( \frac{c}{c+a} \right )^3=\frac{1}{\left ( 1+\frac{b}{a} \right )^3}+\frac{1}{\left ( 1+\frac{c}{b} \right )^3}+\frac{1}{\left ( 1+\frac{a}{c} \right )^3}$

    $=\frac{1}{(1+x)^3}+\frac{1}{(1+y)^3}+\frac{1}{(1+z)^3}$       Với  $\frac{b}{a}=x,\frac{c}{b}=y,\frac{a}{c}=z\rightarrow xyz=1$

Sử dụng bất đẳng thức Am-GM:

    $\frac{1}{(1+x)^3}+\frac{1}{(1+x)^3}+\frac{1}{8}\geq \frac{3}{2}.\frac{1}{(1+x)^2}\rightarrow \frac{1}{(1+x)^3}\geq \frac{3}{4}.\frac{1}{(1+x)^2}-\frac{1}{16}$

    $\rightarrow P\geq \frac{3}{4}.\left [ \frac{1}{(1+x)^2}+\frac{1}{(1+y)^2}+ \frac{1}{(1+z)^2}\right ]-\frac{3}{16}$

Ta cần chứng minh :  

            $\frac{1}{(1+x)^2}+\frac{1}{(1+y)^2}+ \frac{1}{(1+z)^2}\geq \frac{3}{4}$    (*)

Ta có bài toán quen thuộc:

Với  $m,n,p,q$  là các số dương có tích bằng 1 thì  

            $\frac{1}{(1+m)^2}+ \frac{1}{(1+n)^2}+\frac{1}{(1+p)^2}+ \frac{1}{(1+q)^2}\geq 1$

Áp dụng vào bài toán với  $m=x,n=y,p=z,q=1$ ta có  (*)

Vậy bài toán được chứng minh xong

Áp dụng: $\dfrac{1}{(x+1)^2}+\dfrac{1}{(y+1)^2}\geqslant \dfrac{1}{xy+1}$




#568323 Topic tổng hợp một số bất đẳng thức trong kì thi MO các nước

Đã gửi bởi dogsteven on 26-06-2015 - 16:39 trong Bất đẳng thức - Cực trị

Bài 155:(Slovenia TST): Cho a,b,c>0. CMR: $a^2+b^2+c^2=3$ thì

$\sum \frac{a}{b+2}\leq 1$

Bất đẳng thức trên tương đương với: $ab^2+bc^2+ca^2-abc\leqslant 2$

Giả sử $a=\text{median}\{b,c\}$ thì $bc^2+ca^2-a(c^2+bc)=c(a-c)(a-b)\leqslant 0$ nên $ab^2+bc^2+ca^2-abc\leqslant a(b^2+c^2)=a(3-a^2)\leqslant \dfrac{[4+(a-1)^2]^2}{8}\leqslant 2$ do $0\leqslant a\leqslant \sqrt{3}$

Từ đó suy ra điều phải chứng minh. Đẳng thức xảy ra khi và chỉ khi $a=b=c=1$




#566882 Topic tổng hợp một số bất đẳng thức trong kì thi MO các nước

Đã gửi bởi dogsteven on 19-06-2015 - 16:25 trong Bất đẳng thức - Cực trị

Xin trình bày lời giải bài số 119 như sau:

 

Không mất tính tổng quát ta giả sử rằng $a\geq b\geq c$

 

khi đó ta dễ thấy rằng: $a^2\geq b^2\geq c^2$ ; $\frac{1}{b^2+c^2+bc}\geq \frac{1}{c^2+ca+a^2}\geq \frac{1}{a^2+ab+b^2}$

 

Áp dụng bất đẳng thức Chebysev,và bất đẳng thức AM-GM ta có:

$\frac{a^2}{b^2+bc+c^2}+\frac{b^2}{c^2+ca+a^2}+\frac{c^2}{a^2+ab+b^2}\geq \frac{1}{3}(a^2+b^2+c^2)(\sum \frac{1}{b^2+bc+c^2})\geq 3(a^2+b^2+c^2)(\frac{1}{2(a^2+b^2+c^2)+ab+bc+ca})\geq 3(a^2+b^2+c^2)(\frac{1}{3(a^2+b^2+c^2)})=1$

 

Vậy là ta có đpcm. :ukliam2:  :ukliam2:  :ukliam2: 

Áp dụng bất đẳng thức Cauchy-Schwarz:

$\sum \dfrac{a^2}{b^2+c^2+bc}\geqslant \dfrac{(a^2+b^2+c^2)^2}{2(a^2b^2+b^2c^2+c^2a^2)+abc(ab+b+c)}$

Mặt khác $(a^2+b^2+c^2)^2\geqslant 3(a^2b^2+b^2c^2+c^2a^2)\geqslant 3abc(a+b+c)$ nên

$\dfrac{(a^2+b^2+c^2)^2}{2(a^2b^2+b^2c^2+c^2a^2)+abc(ab+b+c)}\geqslant \dfrac{(a^2+b^2+c^2)^2}{\dfrac{2}{3}(a^2+b^2+c^2)^2+\dfrac{1}{3}(a^2+b^2+c^2)^2}=1$




#566839 Topic tổng hợp một số bất đẳng thức trong kì thi MO các nước

Đã gửi bởi dogsteven on 19-06-2015 - 13:00 trong Bất đẳng thức - Cực trị

Bài 115. 

Bất đẳng thức đã cho tương đương với: $\sum \left[\dfrac{1}{yz}-\dfrac{3}{(x+y)(x+z)}\right](y-z)^2\geqslant 0$

(1) $(y+z)(y+x)-3zx=y^2+y(z+x)-2zx=y(y+z)+xy-2zx\geqslant 2x(y-z)\geqslant 0$

(2) $\dfrac{1}{yz}-\dfrac{3}{(x+y)(x+z)}\geqslant \dfrac{1}{4yz}\geqslant 0$

(3) $\dfrac{1}{zx}+\dfrac{1}{xy}-\dfrac{3}{(y+z)(y+x)}-\dfrac{3}{(z+x)(z+y)}=\dfrac{x^2(y^2+z^2)+x(y^3+z^3)+yz(y+z)^2-4x^2yz}{xyz(x+y)(y+z)(z+x)}$

Mà $x^2(y^2+z^2)+x(y^3+z^3)+yz(y+z)^2-4x^2yz\geqslant x(y^3+z^3)-x^2yz\geqslant yz(y+z-x)\geqslant 0$

nên $\dfrac{1}{zx}+\dfrac{1}{xy}-\dfrac{3}{(y+z)(y+x)}-\dfrac{3}{(z+x)(z+y)}\geqslant 0$

Theo tiêu chuẩn S.O.S ta có điều phải chứng minh.




#564473 Topic tổng hợp một số bất đẳng thức trong kì thi MO các nước

Đã gửi bởi dogsteven on 08-06-2015 - 20:18 trong Bất đẳng thức - Cực trị

 

Bài 62:
Trường hợp 1:$min{\{a,b,c}\} \geq -\frac{3}{4} \Rightarrow a,b,c \geq -\frac{3}{4}$
Ta cần chứng minh $\frac{a}{a^2+1} \leq \frac{36a+3}{50}$ (do xài pp tiếp tuyến nên biết cần cm cái này)
$ \Leftrightarrow \frac{(4a+3)(3a-1)^2}{50(a^2+1)} \geq 0 $
Tương tự ta được: $ \sum\frac{a}{a^2+1} \leq \frac{9}{10}$
Trường hợp 2:$min{\{a,b,c}\} < -\frac{3}{4} $
Giả sử $ c=min{\{a,b,c}\} \Rightarrow c<\frac{3}{4} $
Nếu $ a \geq 0 , b \leq 0 $ thì  $\frac{b}{b^2+1}+\frac{c}{c^2+1} <0$ và $\frac{a}{a^2+1} \leq \frac{1}{2}$ nên $VT < \frac{1}{2} < \frac{9}{10}$
Tương tự với $a \leq 0;b \geq 0$
Nếu $a \leq 0;b \leq 0$ thì $VT<0 < \frac{9}{10} $
Nếu $a \geq 0;b \geq 0$ thì ta có:

  1. Nếu $a \in [0;\frac{1}{2}] \cup [2;+\infty)$ thì $ \frac{a}{a^2+1} \leq \frac{2}{5} $.Kết hợp với $\frac{b}{b^2+1} \leq \frac{1}{2} ; \frac{c}{c^2+1} <0$ ta được $VT < \frac{9}{10}$
  2. Nếu $a \in [\frac{1}{2};2] $.
    -Xét $b \in [0;\frac{1}{2}] \cup [2;+\infty)$ thì  $ \frac{b}{b^2+1} \leq \frac{2}{5} $
    Kết hợp với $\frac{a}{a^2+1} \leq \frac{1}{2} ; \frac{c}{c^2+1} <0$ ta được $VT < \frac{9}{10}$
    -Xét $b \in [\frac{1}{2};2]$ thì ta có:$c=1-a-b \geq -3$
    Do $c \in [-3;-\frac{3}{4})$ nên $\frac{c}{c^2+1} < -\frac{1}{10}$
    Do đó ta có:$VT<\frac{1}{2}+\frac{1}{2}-\frac{1}{10}=\frac{9}{10} $.
    Vậy $\sum\frac{a}{a^2+1} \leq \frac{9}{10} $.Dấu "=" xảy ra khi $a=b=c=\frac{1}{3}$

 

Bất đẳng thức tương đương với: $\sum \dfrac{(a-1)^2}{a^2+1}\geqslant \dfrac{6}{5}$

Giả sử $(3b-1)(3c-1)\geqslant 0\Leftrightarrow b^2+c^2\leqslant \dfrac{1}{9}+\left(\dfrac{2}{3}-a\right)^2$

Đến đây ta có $\dfrac{(b-1)^2}{b^2+1}+\dfrac{(c-1)^2}{c^2+1}\geqslant \dfrac{9(a+1)^2}{19+\left(2-3a\right)^2}$

Đến đây biến đổi tương đương.




#565669 Topic tổng hợp một số bất đẳng thức trong kì thi MO các nước

Đã gửi bởi dogsteven on 14-06-2015 - 12:37 trong Bất đẳng thức - Cực trị

Bài 105: (Japanese 1997) Cho $a,b,c$ là những số thực dương.Chứng minh rằng $\frac{(b+c-a)^{2}}{a^{2}+(b+c)^{2}}+\frac{(a+c-b)^{2}}{b^{2}+(a+c)^{2}}+\frac{(b+a-c)^{2}}{c^{2}+(b+a)^{2}}\geq \frac{3}{5}$

Bất đẳng thức trên tương đương với: $\sum \dfrac{a(b+c)}{a^2+(b+c)^2}\leqslant \dfrac{6}{5}$

Áp dụng bất đẳng thức Cauchy-Schwarz: $a^2+(b+c)^2\geqslant a(b+c)+\dfrac{3(b+c)^2}{4}$

Do đó ta có $\sum \dfrac{a(b+c)}{a^2+(b+c)^2}\leqslant \sum \dfrac{4a(b+c)}{4a(b+c)+3(b+c)^2}=3-\sum \dfrac{3(b+c)^2}{4a(b+c)+3(b+c)^2}$

Áp dụng bất đẳng thức Cauchy-Schwarz: $\sum \dfrac{(b+c)^2}{4a(b+c)+3(b+c)^2}\geqslant \dfrac{2(a+b+c)^2}{3(a^2+b^2+c^2)+7(ab+bc+ca)}\geqslant \dfrac{3}{5}$

Do đó $3-\sum \dfrac{3(b+c)^2}{4a(b+c)+3(b+c)^2}\leqslant \dfrac{6}{5}$, đây là điều ta cần chứng minh.




#564471 Topic tổng hợp một số bất đẳng thức trong kì thi MO các nước

Đã gửi bởi dogsteven on 08-06-2015 - 20:11 trong Bất đẳng thức - Cực trị

Bài 59:(PP điểm rơi trong AM-GM)(VNTST 2001)

Cho các số thực dương $a,b,c$ thỏa mãn: $2x+4y+7z=2xyz$. Tìm giá trị nhỏ nhất của biểu thức: $P=x+y+z$

 

p/s: trình bày luôn cách tìm điểm rơi nhé!

Ta dùng nhân tử để tìm điểm rơi.

Xét hàm số $L=x+y+z+\lambda(2x+4y+7z-2xyz)$

Đạo hàm riêng cho biến $x$ được $1=2\lambda (yz-1)$

Đạo hàm riêng cho biến $y$ được $1=2\lambda(zx-2)$

Đạo hàm riêng cho biến $z$ được $1=\lambda(2xy-7)$

Do đó $2(yz-1)=2(zx-2)=2xy-7$




#564469 Topic tổng hợp một số bất đẳng thức trong kì thi MO các nước

Đã gửi bởi dogsteven on 08-06-2015 - 20:07 trong Bất đẳng thức - Cực trị

 Bất đẳng thức đồng bậc 3 nên ta chuẩn hoá $a^2+b^2+c^2=9$

 Khi đó : BĐT $\Leftrightarrow 2(a+b+c)-abc\leq 10\Leftrightarrow a(2-bc)+(b+c).2\leq 10$

 Giả sử $a\geq b\geq c \Rightarrow |bc|\leq \frac{9-a^2}{2}\leq 3 \Rightarrow bc\in [-3;3]$

 Áp dụng Cauchy-Schwarz ta có : $a(2-bc)+(b+c).2\leq \sqrt{(9+2bc)(b^2c^2-4bc+8)}\leq 10\Leftrightarrow (bc+2)^2(2bc-7)\leq 0$

 Luôn đúng $\forall ~bc\in [-3;3]$

 Từ đó có đpcm

Ta có một lời giải rất xấu bằng phân tách trường hợp như sau:

Chuẩn hóa $a^2+b^2+c^2=9$. Giả sử $a\geqslant b\geqslant c$

Nếu $a\leqslant 0$ thì bất đẳng thức hiển nhiên đúng do $abc>-10$

Nếu $a\geqslant 0$ và $b,c\leqslant 0$ thì $2(a+b+c)\leqslant 2a\leqslant 6\leqslant 10\leqslant 10+abc$

Nếu $a\geqslant b\geqslant 0\geqslant c$ thì $2a+2b+2c\le 9+3c$ nên ta cần chứng minh $3c\le 1+\dfrac{c(9-c^2)}{2}$ luôn đúng.

Nếu $c\geqslant 0$ thì $2a+2b+2c\leqslant 9+c$

- Trường hợp $c\leqslant 1$ thì $9+c\leqslant 10\leqslant 10+abc$

- Trường hợp $c\geqslant 1$ thì $9+c\leqslant 9+abc\leqslant 10+abc$




#560970 Topic tổng hợp một số bất đẳng thức trong kì thi MO các nước

Đã gửi bởi dogsteven on 22-05-2015 - 19:54 trong Bất đẳng thức - Cực trị

Bài 41. 

Giả sử $x=\text{min}\{x,y,z\}$, Thay trực tiếp $x=\dfrac{4-yz}{y+z+yz}$ ta được:

$VT-VP=y^2+z^2+\dfrac{(4-yz)^2}{y+z+yz}-2\left(1+\sqrt{2}\right)\left(y+z+\dfrac{4-yz}{y+z+yz}\right)+3+6\sqrt{2}$

Đặt $a=y+z$ và $b=yz$ thì ta có $f(b)=a^2-2b+\dfrac{(4-b)^2}{(a+b)^2}-2(1+\sqrt{2})\left(a+\dfrac{4-b}{a+b}\right)+3+6\sqrt{2}$

$=-2b+\dfrac{4-b}{(a+b)^2}\left[4-b-2(1+\sqrt{2})(a+b)\right]+a^2-2(1+\sqrt{2})a+3+6\sqrt{2}$

Có:

$4-b-2(1+\sqrt{2})(a+b)\geqslant 4-\dfrac{a^2}{4}-2(1+\sqrt{2})\left(a+\dfrac{a^2}{4}\right)$

$4-b\geqslant 4-\dfrac{a^2}{4}$

$-2b\geqslant \dfrac{a^2}{2}$

$\dfrac{1}{(a+b)^2}\geqslant \dfrac{1}{\left(a+\dfrac{a^2}{4}\right)^2}$

Do đó ta chỉ cần chứng minh khi $b=\dfrac{a^2}{4}$




#565745 Topic tổng hợp một số bất đẳng thức trong kì thi MO các nước

Đã gửi bởi dogsteven on 14-06-2015 - 20:58 trong Bất đẳng thức - Cực trị

 Nếu thay $a=1/3$ vào thì $k(a-1/3)=0$ như vậy là mất k rồi làm sao tìm được anh ?

Nhân tử lại đã. Rồi rút gọn cái $a-\dfrac{1}{3}$, sau đó mới cho $a=\dfrac{1}{3}$




#565749 Topic tổng hợp một số bất đẳng thức trong kì thi MO các nước

Đã gửi bởi dogsteven on 14-06-2015 - 21:04 trong Bất đẳng thức - Cực trị

Tổng quát cách làm:

Với bất đẳng thức có dạng tổng hàm $f(x)+f(y)+f(z)\geqslant 0$ với điều kiện $x+y+z=3$ ta có thể sẽ chứng minh: $f(x)\geqslant f'(1)(x-1)+f(1)$

Với bất đẳng thức có dạng tổng hàm $f(x)+f(y)+f(z)\geqslant 0$ với điều kiện $xyz=1$ thì ta có thể sẽ chứng minh: $f(x)\geqslant f'(x)\ln x+f(1)$




#566836 Topic tổng hợp một số bất đẳng thức trong kì thi MO các nước

Đã gửi bởi dogsteven on 19-06-2015 - 12:37 trong Bất đẳng thức - Cực trị

Có vẽ đề bị thiếu thì phải?  :closedeyes:

Chắc là chứng minh nó lớn hơn hoặc bằng $\dfrac{3}{8}$ :))




#566364 Topic tổng hợp một số bất đẳng thức trong kì thi MO các nước

Đã gửi bởi dogsteven on 17-06-2015 - 09:46 trong Bất đẳng thức - Cực trị

Bài xx (IMO 2006). Tìm hằng số $M$ nhỏ nhất sao cho  bất đẳng thức

\[\left | ab(a^2-b^2)+bc(b^2-c^2)+ca(c^2-a^2) \right | \leqslant M(a^2+b^2+c^2)^2,\]

luôn đúng với mọi số thực $a,\,b,\,c$ thay đổi tùy ý.

Đặt $p=a+b+c, q=ab+bc+ca, r=abc$. Không mất tính tổng quát, chuẩn hóa $p=1$. Khi đó bất đẳng thức trở thành:

$q^2-4q^3+2(9q-2)r-27r^2\leqslant M^2(1-2q)^4$

Cố định $q$, ta xét hàm số vế trái theo $r$, do đó hàm số này đạt cực đại tại $r=\dfrac{9q-2}{27}$, do đó ta cần phải có: $M^2\geqslant \dfrac{4(1-3q)^3}{27(1-2q)^4}=f(q)$

Để $M^2\geqslant f(q)$ với mọi $q$ khi và chỉ khi $M^2\geqslant \text{max}f(q)$

Ta có $f'(q)=\dfrac{4(1+6q)(1-3q)^2}{27(2q-1)^5}=0\Leftrightarrow q=\dfrac{1}{3}$ hoặc $q=\dfrac{-1}{6}$

Giờ thế giá trị ta được $M\geqslant \dfrac{9\sqrt{2}}{32}$




#565761 Topic tổng hợp một số bất đẳng thức trong kì thi MO các nước

Đã gửi bởi dogsteven on 14-06-2015 - 21:21 trong Bất đẳng thức - Cực trị

chú em dogsteven học với tốc độ thế này thì quá tốt,hy vọng được nghe tin tốt lành từ chú em ở VMO 2 năm nữa :))

( hơi spam )

Em định không thi VMO, mấy năm học chuyên em học lập trình game với photoshop :)) (đam mê)

(hơi spam)




#565758 Topic tổng hợp một số bất đẳng thức trong kì thi MO các nước

Đã gửi bởi dogsteven on 14-06-2015 - 21:17 trong Bất đẳng thức - Cực trị

Giờ đã học hết cả chương trình PT thế này rồi à em. Anh giờ còn chưa biết mấy cái $In$ này là gì :unsure:

Em học cho vui.

P.s. Đây có phải spam không nhỉ :))




#559479 Topic tổng hợp một số bất đẳng thức trong kì thi MO các nước

Đã gửi bởi dogsteven on 15-05-2015 - 10:04 trong Bất đẳng thức - Cực trị

Bài 6. $\sum a . \sum \dfrac{1}{a} - 9 =\sum \dfrac{(b-c)^2}{bc}\geqslant 3\sqrt[3]{\dfrac{\prod (b-c)^2}{(abc)^2}}$